Find k for this probability density function to be valid.

In summary, The student attempted to find a solution to a homework equation in which the integral from -infty to +infty was 1. They found that the solution was 1/theta when t=-\frac{x-μ}{\theta}.
  • #1
TelusPig
15
0

Homework Statement



Find k such that the function [itex]f(x)=ke^{-\frac{x-\mu}{\theta}}[/itex] is a probability density function (pdf), for [itex] x > \mu, \mu[/itex] and [itex]\theta[/itex] are constant.

Homework Equations



The property of a pdf says that the integral of f(x) from [itex]-\infty[/itex] to [itex]\infty[/itex] equals 1, that is [itex]\int\limits_{-\infty}^\infty f(x)dx=1[/itex]

The Attempt at a Solution



[itex]\int\limits_{-\infty}^\infty ke^{-\frac{x-\mu}{\theta}}dx[/itex]
[itex]=k\int\limits_{-\infty}^\infty e^{-\frac{x-\mu}{\theta}}dx[/itex]
Let [itex]t=-\frac{x-\mu}{\theta} => -\theta dt=dx[/itex]

[itex]=> -k\int\limits_\infty^{-\infty} e^t(-\theta)dt[/itex]

[itex]=>k\theta\int\limits_{-\infty}^\infty e^tdt[/itex]

But [itex]e^t[/itex] would diverge going towards infinity? How could this integral be equal to 1. I am not sure what I'm doing wrong.
 
Physics news on Phys.org
  • #2
TelusPig said:

Homework Statement



Find k such that the function [itex]f(x)=ke^{-\frac{x-\mu}{\theta}}[/itex] is a probability density function (pdf), for [itex] x > \mu, \mu[/itex] and [itex]\theta[/itex] are constant.

Homework Equations



The property of a pdf says that the integral of f(x) from [itex]-\infty[/itex] to [itex]\infty[/itex] equals 1, that is [itex]\int\limits_{-\infty}^\infty f(x)dx=1[/itex]

The Attempt at a Solution



[itex]\int\limits_{-\infty}^\infty ke^{-\frac{x-\mu}{\theta}}dx[/itex]
[itex]=k\int\limits_{-\infty}^\infty e^{-\frac{x-\mu}{\theta}}dx[/itex]
Let [itex]t=-\frac{x-\mu}{\theta} => -\theta dt=dx[/itex]

[itex]=> -k\int\limits_\infty^{-\infty} e^t(-\theta)dt[/itex]

[itex]=>k\theta\int\limits_{-\infty}^\infty e^tdt[/itex]

But [itex]e^t[/itex] would diverge going towards infinity? How could this integral be equal to 1. I am not sure what I'm doing wrong.

It's supposed to be a density function only for x>μ. You don't want to integrate from -infinity to +infinity. You want to integrate from μ to +infinity. The pdf will be zero for x<μ.
 
  • #3
Ok I see what you mean. But even if I do that, the upper bound of infinity is giving me problems because e^(infinity) is infinity :S
 
  • #4
Never mind! I finally figured it out... I had a -infinity instead. All is good :) I got k = 1/theta if anyone was interested
 

Related to Find k for this probability density function to be valid.

1. What is the purpose of finding k for a probability density function?

The value of k determines the overall shape and range of a probability density function. It helps to ensure that the function accurately represents the probability of different outcomes occurring.

2. How is k calculated for a probability density function?

K is typically calculated by integrating the probability density function over its entire range and setting the result equal to 1. This allows for the total probability of all possible outcomes to equal 1.

3. Can k have a negative value?

No, k must be a positive value in order for the probability density function to be valid. A negative value for k would result in negative probabilities, which do not make sense in a probability distribution.

4. What happens if k is set to a value other than 1?

If k is set to a value other than 1, it will affect the overall shape and range of the probability density function. This means that the probabilities for different outcomes will also be adjusted accordingly.

5. Is finding k the only step in creating a valid probability density function?

No, finding k is just one step in creating a valid probability density function. Other important steps include ensuring that the function is continuous, non-negative, and that the total area under the curve is equal to 1.

Similar threads

  • Calculus and Beyond Homework Help
Replies
4
Views
436
  • Calculus and Beyond Homework Help
Replies
4
Views
576
  • Calculus and Beyond Homework Help
Replies
2
Views
423
  • Calculus and Beyond Homework Help
Replies
19
Views
2K
  • Calculus and Beyond Homework Help
Replies
14
Views
2K
  • Calculus and Beyond Homework Help
Replies
1
Views
1K
  • Calculus and Beyond Homework Help
Replies
3
Views
732
  • Calculus and Beyond Homework Help
Replies
3
Views
616
  • Calculus and Beyond Homework Help
Replies
3
Views
617
  • Calculus and Beyond Homework Help
Replies
31
Views
3K
Back
Top